Những câu hỏi liên quan
Lê Tài Bảo Châu
Xem chi tiết
Tran Le Khanh Linh
2 tháng 6 2020 lúc 21:19

BĐT tương đương : \(\frac{a\left(a+c+b-3b\right)}{1+ab}+\frac{b\left(b+a+c-3c\right)}{a+bc}+\frac{c\left(c+b+a-3a\right)}{1+ca}\ge0\)

\(\Leftrightarrow\frac{3a\left(1-b\right)}{1+ab}+\frac{3b\left(1-c\right)}{1+bc}+\frac{3c\left(1-a\right)}{1+ca}\ge0\)

\(\Leftrightarrow\frac{a\left(1-b\right)}{1+ab}+\frac{b\left(1-c\right)}{1+bc}+\frac{c\left(1-a\right)}{1+ca}\ge0\)

\(\Leftrightarrow\frac{a\left(1-b\right)}{1+ab}+1+\frac{b\left(1-c\right)}{1+bc}+1+\frac{c\left(1-a\right)}{1+ca}\ge3\)

\(\Leftrightarrow\frac{a+1}{1+ab}+\frac{b+1}{1+bc}+\frac{c+1}{1+ca}\ge3\)

Áp dụng BĐT Cosi ta có: \(\frac{a+1}{1+ab}+\frac{b+1}{1+bc}+\frac{c+1}{1+ca}\ge3\sqrt[3]{\frac{a+1}{1+ab}\cdot\frac{b+1}{1+bc}\cdot\frac{c+1}{1+ca}}\)

Ta phải chứng minh: \(\sqrt[3]{\frac{a+1}{1+ab}\cdot\frac{b+1}{1+bc}\cdot\frac{c+1}{1+ca}}\ge1\)

\(\Leftrightarrow\left(a+1\right)\left(b+1\right)\left(c+1\right)\ge\left(1+ab\right)\left(1+bc\right)\left(1+ca\right)\)

Thật vậy \(\left(a+1\right)\left(b+1\right)\left(c+1\right)\ge\left(1+ab\right)\left(1+bc\right)\left(1+ca\right)\)

\(\Leftrightarrow abc+ab+bc+ca+a+b+c+1\ge a^2b^2c^2+abc\left(a+b+c\right)+ab+bc+ca+1\)

\(\Leftrightarrow3\ge a^2b^2c^2+2abc\) (*)

Từ a+b+c=3 => \(3\ge3\sqrt[3]{abc}\Leftrightarrow abc\le1\)

=> (*) đúng

Vậy \(\frac{a\left(a+c-2b\right)}{1+ab}+\frac{b\left(b+a-2c\right)}{1+bc}+\frac{c\left(c+b-2a\right)}{1+ca}\ge0\)

Đẳng thức xảy ra <=> a=b=c=1

Khách vãng lai đã xóa
zZz Cool Kid_new zZz
5 tháng 6 2020 lúc 14:32

đay nha

Khách vãng lai đã xóa
Kuramajiva
Xem chi tiết
Nguyễn Việt Lâm
30 tháng 12 2020 lúc 21:34

1. Đề thiếu

2. BĐT cần chứng minh tương đương:

\(a^4+b^4+c^4\ge abc\left(a+b+c\right)\)

Ta có:

\(a^4+b^4+c^4\ge\dfrac{1}{3}\left(a^2+b^2+c^2\right)^2\ge\dfrac{1}{3}\left(ab+bc+ca\right)^2\ge\dfrac{1}{3}.3abc\left(a+b+c\right)\) (đpcm)

3.

Ta có:

\(\left(a^6+b^6+1\right)\left(1+1+1\right)\ge\left(a^3+b^3+1\right)^2\)

\(\Rightarrow VT\ge\dfrac{1}{\sqrt{3}}\left(a^3+b^3+1+b^3+c^3+1+c^3+a^3+1\right)\)

\(VT\ge\sqrt{3}+\dfrac{2}{\sqrt{3}}\left(a^3+b^3+c^3\right)\)

Lại có:

\(a^3+b^3+1\ge3ab\) ; \(b^3+c^3+1\ge3bc\) ; \(c^3+a^3+1\ge3ca\)

\(\Rightarrow2\left(a^3+b^3+c^3\right)+3\ge3\left(ab+bc+ca\right)=9\)

\(\Rightarrow a^3+b^3+c^3\ge3\)

\(\Rightarrow VT\ge\sqrt{3}+\dfrac{6}{\sqrt{3}}=3\sqrt{3}\)

Nguyễn Việt Lâm
30 tháng 12 2020 lúc 21:37

4.

Ta có:

\(a^3+1+1\ge3a\) ; \(b^3+1+1\ge3b\) ; \(c^3+1+1\ge3c\)

\(\Rightarrow a^3+b^3+c^3+6\ge3\left(a+b+c\right)=9\)

\(\Rightarrow a^3+b^3+c^3\ge3\)

5.

Ta có:

\(\dfrac{a}{b}+\dfrac{b}{c}\ge2\sqrt{\dfrac{a}{c}}\) ; \(\dfrac{a}{b}+\dfrac{c}{a}\ge2\sqrt{\dfrac{c}{b}}\) ; \(\dfrac{b}{c}+\dfrac{c}{a}\ge2\sqrt{\dfrac{b}{a}}\)

\(\Rightarrow\sqrt{\dfrac{b}{a}}+\sqrt{\dfrac{c}{b}}+\sqrt{\dfrac{a}{c}}\le\dfrac{a}{b}+\dfrac{b}{c}+\dfrac{c}{a}=1\)

Nguyễn Việt Lâm
30 tháng 12 2020 lúc 21:39

Câu 1:

\(VT=1-\dfrac{1}{2}+\dfrac{1}{2}-\dfrac{1}{3}+...+\dfrac{1}{n-1}-\dfrac{1}{n}\)

\(VT=1-\dfrac{1}{n}< 1\) (đpcm)

Trần Nguyễn Ngọc Hưng
Xem chi tiết
Nguyễn Đăng Nhân
21 tháng 2 2022 lúc 17:57

Ta đổi chiều bất đẳng thức, khi đó bất đẳng thức cần chứng minh tương đương với:

\(18\left(\frac{a^3}{1+a^3}+\frac{b^3}{1+b^3}+\frac{c^3}{1+c^3}\right)+\left(a+b+c\right)^3\ge54\)

Để ý abc=1 thì \(\frac{a^3}{1+a^3}=\frac{a^3}{abc+a^3}=\frac{a^2}{bc+a^2}\)nên bất đẳng thức trên thành:

\(18\left(\frac{a^2}{bc+a^2}+\frac{b^2}{ca+b^2}+\frac{c^2}{ab+c^2}\right)+\left(a+b+c\right)^3\ge54\)

Lại cũng từ \(abc=1\) ta có \(\left(a+b+c\right)^3\ge27abc=27\), do đó ta sẽ chứng minh được khi ta chỉ ra được:

\(\frac{a^2}{bc+a^2}+\frac{b^2}{ca+b^2}+\frac{c^2}{ab+c^2}\ge\frac{3}{2}\)

Vế trái của đánh giá trên áp dụng bất đẳng thức Bunhiacopxki dạng phân thức. Lúc này ta được:

\(\frac{a^2}{bc+a^2}+\frac{b^2}{ca+b^2}+\frac{c^2}{ab+c^2}\ge\frac{\left(a+b+c\right)^2}{a^2+b^2+c^2+ab+bc+ca}\)

Tuy nhiên để đến khi \(a=b=c=1\) thì:

\(\frac{18\left(a+b+c\right)^2}{a^2+b^2+c^2+ab+bc+ca}=\left(a+b+c\right)^3=27\)

Ta sử dụng bất đẳng thức Cauchy dạng \(x+y\ge2\sqrt{xy}\), khi đó ta được:

\(\frac{18\left(a+b+c\right)^2}{a^2+b^2+c^2+ab+bc+ca}+\left(a+b+c\right)^3\ge\sqrt{\frac{18\left(a+b+c\right)^5}{a^2+b^2+c^2+ab+bc+ca}}\)

Chứng minh sẽ hoàn tất nếu ta chỉ được:

\(\sqrt{\frac{18\left(a+b+c\right)^5}{a^2+b^2+c^2+ab+bc+ca}}\ge54\Leftrightarrow\left(a+b+c\right)^5\ge\frac{81}{2}\left(a^2+b^2+c^2+ab+bc+ca\right)\)

Vậy theo bất đẳng thức Cauchy ta được:

\(\left(a+b+c\right)^6=\left[\left(a^2+b^2+c^2\right)+\left(ab+bc+ca\right)+\left(ab+bc+ca\right)\right]^3\)

\(\ge27\left(a+b+c\right)^2\left(ab+bc+ca\right)^2\ge81abc\left(a^2+b^2+c^2\right)\left(a+b+c\right)\)

\(=81\left(a^2+b^2+c^2\right)\left(a+b+c\right)\)

Khi đó ta được:

\(\left(a+b+c\right)^5\ge81\left(a^2+b^2+c^2\right)\)

Vậy ta cần chỉ ra rằng:

\(2\left(a^2+b^2+c^2\right)\ge a^2+b^2+c^2+ab+bc+ca\)

Vậy bất đẳng thức trên tương đương với \(\left(a-b\right)^2+\left(b-c\right)^2+\left(c-a\right)^2\ge0\), là một bất đẳng thức hiển nhiên đúng.

Vậy bất đẳng thức được chứng minh, dấu đẳng thức xảy ra khi \(a=b=c=1\)

Khách vãng lai đã xóa
Kiệt Nguyễn
Xem chi tiết
Phùng Minh Quân
24 tháng 4 2020 lúc 17:04

\(\Sigma\frac{a^3+1}{b^3+c^3+1}=(\frac{-\left(a+b\right)\left(c^3+1\right)}{ab\left(a+b+c\right)\left(a^3+b^3+1\right)}+\frac{\Sigma\left(a+b\right)^2}{3\left(a+b+c\right)}+2\left(a+b+c\right)\)

\(+\frac{\frac{1}{2}\left(\sqrt[3]{a}+\sqrt[3]{b}+\sqrt[3]{c}\right)\left(\Sigma\frac{1}{\sqrt[3]{a^2}+\sqrt[3]{ab}+\sqrt[3]{b^2}}\right)}{\left(a+b+c\right)^2+3\left(a+b+c\right)+9}+\frac{\Sigma\left(a-b\right)^2}{a+b+c})\left(a-b\right)^2+2\ge2\)

justforfun:) 

Khách vãng lai đã xóa
Nguyễn Văn Duy
Xem chi tiết
Trà My
27 tháng 2 2017 lúc 22:28

\(abc=1\Rightarrow\left(abc\right)^2=a^2b^2c^2=1\Rightarrow a^2=\frac{1}{b^2c^2}\Rightarrow\frac{1}{a^3\left(b+c\right)}=\frac{b^2c^2}{a\left(b+c\right)}=\frac{\left(bc\right)^2}{ab+ac}\)

Chứng minh tương tự ta có:  \(\frac{1}{b^3\left(c+a\right)}=\frac{\left(ca\right)^2}{bc+ba};\frac{1}{c^3\left(a+b\right)}=\frac{\left(ab\right)^2}{ca+cb}\)

=> \(\frac{1}{a^3\left(b+c\right)}+\frac{1}{b^3\left(c+a\right)}+\frac{1}{c^3\left(a+b\right)}=\frac{\left(ab\right)^2}{bc+ca}+\frac{\left(bc\right)^2}{ab+ca}+\frac{\left(ca\right)^2}{ab+bc}\)

Áp dụng bđt Cauchy-Schwarz dạng Engel: \(\frac{\left(ab\right)^2}{bc+ca}+\frac{\left(bc\right)^2}{ab+ca}+\frac{\left(ca\right)^2}{ab+bc}\ge\frac{\left(ab+bc+ca\right)^2}{bc+ca+ab+ca+ab+bc}=\frac{ab+bc+ca}{2}\)

Tiếp tục áp dụng bđt Cauchy với 3 số dương ta được: \(\frac{ab+bc+ca}{2}\ge\frac{3\sqrt[3]{ab.bc.ca}}{2}=\frac{3\sqrt[3]{\left(abc\right)^2}}{2}=\frac{3\sqrt[3]{1}}{2}=\frac{3}{2}\)

=> \(\frac{\left(ab\right)^2}{bc+ca}+\frac{\left(bc\right)^2}{ab+ca}+\frac{\left(ca\right)^2}{ab+bc}\ge\frac{ab+bc+ca}{2}\ge\frac{3}{2}\)

Trà My
27 tháng 2 2017 lúc 22:39

Dấu "=" xảy ra khi a=b=c=1

Hoàng Phúc
28 tháng 2 2017 lúc 16:14

viết lại sigma 1/a3(b+c) = sigma 1/a2/a(b+c) 

đến đây dùng schwarz -> sigma 1/a3(b+c) >/ ab+bc+ac/2 >/ 3/2 (AM-GM cho ab,bc,ca)

Dương Thiên Tuệ
Xem chi tiết
Secret
Xem chi tiết
Thắng Nguyễn
23 tháng 5 2016 lúc 12:42

Áp dụng BĐT AM-GM ta có:

\(\frac{a}{c}+\frac{a}{c}+\frac{b}{a}\ge3\sqrt[3]{\frac{abc}{c^3}}=\frac{3}{c}\left(1\right)\)

Chứng minh tương tự ta cũng có:

\(\frac{c}{b}+\frac{c}{b}+\frac{a}{c}\ge\frac{3}{b}.\left(2\right)\frac{b}{a}+\frac{b}{a}+\frac{c}{a}\ge\frac{3}{a}.\left(3\right)\)

Cộng theo vế của (1),(2) và (3) ta được

\(3\left(\frac{b}{a}+\frac{a}{c}+\frac{c}{b}\right)\ge3\left(\frac{1}{a}+\frac{1}{b}+\frac{1}{c}\right).\left(4\right)\)

Mặt khác, do abc=1 nên theo BĐT AM-GM 

ta có \(\frac{a}{b}+\frac{b}{c}+\frac{c}{a}+\frac{1}{a}+\frac{1}{b}+\frac{1}{c}=\left(\frac{a}{b}+ab\right)+\left(\frac{b}{c}+bc\right)+\left(\frac{c}{a}+ca\right)\ge2a+2b+2c.\left(5\right)\)

Từ (4) và (5) =>đpcm

Đẳng thức xảy ra khi a=b=c=1

Thắng Nguyễn
23 tháng 5 2016 lúc 12:08

dùng BĐT AM-GM

Secret
23 tháng 5 2016 lúc 12:44

thanks Nguyễn Huy Thắng

Nguyễn Phương Thảo
Xem chi tiết
Nguyễn Quốc Gia Huy
Xem chi tiết
Thắng Nguyễn
23 tháng 8 2017 lúc 20:20

Bài này làm hoài :v

Áp dụng BĐT Cauchy-Schwarz và BĐT AM-GM ta có:

\(VT=\frac{b^2c^2}{ab+ac}+\frac{a^2c^2}{ab+bc}+\frac{a^2b^2}{ac+bc}\)

\(\ge\frac{\left(ab+bc+ca\right)^2}{2\left(ab+bc+ca\right)}=\frac{ab+bc+ca}{2}\)

\(\ge\frac{3\sqrt[3]{\left(abc\right)^2}}{2}=\frac{3}{2}=VP\)

Khi a=b=c=1

Phan Nghĩa
2 tháng 9 2020 lúc 20:21

Đặt \(\left\{a;b;c\right\}\rightarrow\left\{\frac{1}{x};\frac{1}{y};\frac{1}{z}\right\}\)Khi đó : \(\frac{1}{x}.\frac{1}{y}.\frac{1}{z}=\frac{1}{x.y.z}=a.b.c=1< =>x.y.z=1\)

\(BĐT< =>\frac{1}{\left(\frac{1}{x}\right)^3\left(\frac{1}{y}+\frac{1}{z}\right)}+\frac{1}{\left(\frac{1}{y}\right)^3\left(\frac{1}{y}+\frac{1}{x}\right)}+\frac{1}{\left(\frac{1}{z}\right)^3\left(\frac{1}{x}+\frac{1}{y}\right)}\ge\frac{3}{2}\)

\(< =>\frac{x^3yz}{y+z}+\frac{y^3xz}{z+x}+\frac{z^3xy}{x+y}\ge\frac{3}{2}\)\(< =>\frac{x^2}{y+z}+\frac{y^2}{x+z}+\frac{z^2}{x+y}\ge\frac{3}{2}\)(*)

Ta chỉ cần chỉ ra bất đẳng thức (*) đúng thì bài toán được giải quyết , thật vậy :

Theo bất đẳng thức Bunhiacopxki dạng phân thức :

\(\frac{x^2}{y+z}+\frac{y^2}{z+x}+\frac{z^2}{x+y}\ge\frac{\left(x+y+z\right)^2}{y+z+z+x+x+y}=\frac{\left(x+y+z\right)^2}{2\left(x+y+z\right)}=\frac{x+y+z}{2}\) (**)

Sử dụng bất đẳng thức AM-GM ta có : 

\(x+y+z\ge3\sqrt[3]{xyz}=3\sqrt[3]{1}=3\)Tương đương \(\frac{x+y+z}{2}\ge\frac{3}{2}\)(***)

Từ (**) và (***) ta được \(\frac{x^2}{z+y}+\frac{y^2}{x+z}+\frac{z^2}{x+y}\ge\frac{x+y+z}{2}\ge\frac{3}{2}\)

Suy ra bất đẳng thức (*) đúng . Nên ta có điều phải chứng minh !

Dấu "=" xảy ra khi và chỉ khi \(x=y=z=1< =>a=b=c=1\)

Khách vãng lai đã xóa
Trí Tiên亗
2 tháng 9 2020 lúc 21:43

Bài này dùng Sac - xơ để ý một xíu là ra nhé !

Theo BĐT Svacxo ta có :

\(\frac{1}{a^3\left(b+c\right)}+\frac{1}{b^3\left(c+a\right)}+\frac{1}{c^3\left(a+b\right)}\) \(=\frac{\frac{1}{a^2}}{a\left(b+c\right)}+\frac{\frac{1}{b^2}}{b.\left(c+a\right)}+\frac{\frac{1}{c^2}}{c.\left(a+b\right)}\)

\(\ge\frac{\left(\frac{1}{a}+\frac{1}{b}+\frac{1}{c}\right)^2}{2.\left(ab+bc+ca\right)}=\frac{\left(\frac{ab+bc+ca}{abc}\right)^2}{2.\left(ab+bc+ca\right)}=\frac{\left(ab+bc+ca\right)^2}{2.\left(ab+bc+ca\right)}\)

\(=\frac{ab+bc+ca}{2}\ge\frac{3\sqrt[3]{a^2b^2c^2}}{2}=\frac{3}{2}\) ( Do \(abc=1\) )

Dấu "=" xảy ra \(\Leftrightarrow a=b=c=1\)

Vậy BĐT được hoàn tất !

Khách vãng lai đã xóa